I don’t understand this

I Dont Understand This

Answers

Answer 1

Answer: a=x represents sinks

b=the amount of money he will spend overall on the sink and sealent

c= 100x represents 100 sinks

d=joe will have to charge 85 dollars per sink

Step-by-step explanation:


Related Questions

Help I really don't know how to do this I need help !!!!!!

Answers

Answer:

It would be C for it is the lowest.

Step-by-step explanation:

Since all of these numbers are fractions we can divide them by the denominator and see which one would be the lowest.  If we do that we will end up getting C.

A - 0.5

B - 0.5

C - 0.4

D - 0.45

At an ice cream store, a family ordered 1 banana split and 2 hot fudge sundaes, paying a total of $11 for their order. The next customer ordered 4 banana splits and 3 hot fudge sundaes and paid $29. How much does each item cost?

Answers

Answer:

$5 and $3

Step-by-step explanation:

Cost of banana split = b

Cost of hot fudge sundae = s

b + 2s = 114b + 3s = 29

From the first equation we get:

b = 11 - 2s

Replacing in the second equation:

4(11 - 2s) + 3s = 2944 - 8s + 3s = 2944 - 29 = 5s15 = 5ss = 15/5s = 3

Then

b = 11 - 2*3 = 11 - 6 = 5

So banana split costs $5 and hot fudge sundae costs $3

solve this equation 3/10x-4=40-2x

Answers

I Am Pretty Sure The Answer Is x=440/23

Hope This Help's

Find the distance between the two points. (round to the nearest hundredths)



distance:

Answers

Answer:

d = 5.10

Step-by-step explanation:

The distance between two points is given by

d = sqrt( ( y2-y1)^2 + ( x2-x1)^2 )  where ( x1,y1) and ( x2,y2) are the two points

d = sqrt( ( 5-6)^2 + (8-3)^2 )  

d = sqrt( ( -1)^2 + (5)^2 )  

d = sqrt(  1+25)

d = sqrt(26)

d = 5.099019514

Rounding to the nearest hundredth

d = 5.10

Answer:

[tex]d=\sqrt{26}\approx5.10[/tex]

Step-by-step explanation:

To find the distance between any two points, we can use the distance formula:

[tex]d=\sqrt{(x_2-x_1)^2+(y_2-y_1)^2[/tex]

We can see that Point R is at (3,6), and Point E is at (8,5).

Let's let (3,6) be (x₁, y₁) and let's let (8,5) be (x₂, y₂). So, substituting them into our formula, we'll get:

[tex]d=\sqrt{(8-3)^2+(5-6)^2[/tex]

Subtract:

[tex]d=\sqrt{(5)^2+(-1)^2[/tex]

Square:

[tex]d=\sqrt{25+1}[/tex]

Add:

[tex]d=\sqrt{26}[/tex]

Approximate. Use a calculator:

[tex]d=\sqrt{26}\approx5.10[/tex]

So, the distance between RE is approximately 5.1 units.

And we're done!

What value of x makes the equation true?
147 = 5(1 - 7x) + 2

Answers

Answer: X = -4

Step-by-step explanation:

first you need to solve, so do what needs to be done in the equation, which is just multiply and add

147 = 5 - 35x + 2

147= 7 -35x

get rid of the numbers around x, which would be subtracting 7 from both sides

140 = -35x

x = -4

pleaseeeeeeee help me

Answers

Answer:

by my best guess, x = -5 (negative five)

Step-by-step explanation:

The price to mail a letter at the post office is $0.49 for the first ounce

and $0.21 for each additional ounce. Amrit paid $1.96 to mail his letter.

How much did the letter weigh?

Answers

The letter weighted 7 ounces

∠A and ∠ B ∠B are complementary angles. If m ∠ A = ( 2 x − 10 ) ∘ ∠A=(2x−10) ∘ and m ∠ B = ( x − 2 ) ∘ ∠B=(x−2) ∘ , then find the measure of ∠ A ∠A.

Answers

Answer:

Angle A = 58 degrees

Angle B = 32 degrees

Step-by-step explanation:

First of all, we will need to know that complementary angles are angles that sum up to 90 degrees.

This means that if we add the values of angle A and angle B, they will both sum up to 90 degrees.

i.e (2x-10) + (x-2) = 90

3x -12 = 90

3x = 102

x = 34 degrees

We can substitute this value of x into the equations for angles A and B

A = (2 X (34) - 10 ) = 58 degrees

B =  ( 34 - 2) = 32 degrees

Therefore, angles A and B are 54 and 32 degrees respectively.

Check: We can add the two angles to see if we get 90 degrees. 58 + 32 = 90 degrees. This shows that the two angles are complementary and our answer is correct.

if 18 more than twice a number is -10,what is the number?​

Answers

Answer:

-14

Step-by-step explanation:

Let the number be x

[tex]2x + 18 = - 10 \\ 2x = - 28 \\ x = - 14[/tex]

2x + 18= -10
2x. -18= -10 -18
2x= -28
X=-14

-3 - 3x = -4(2x + 7)

Answers

Answer :    x=−5

Step-by-step explanation: Step 1: Simplify both sides of the equation.

−3−3x=−4(2x+7)

−3+−3x=(−4)(2x)+(−4)(7)(Distribute)

−3+−3x=−8x+−28

−3x−3=−8x−28

Step 2: Add 8x to both sides.

−3x−3+8x=−8x−28+8x

5x−3=−28

Step 3: Add 3 to both sides.

5x−3+3=−28+3

5x=−25

Step 4: Divide both sides by 5.

5x /5 =  −25 /5

x=−5

Answer:

x=−5

How can parallel lines be used to compare figure E and F? Explain your reasoning.

Answers

When the e and f are displayed

Write -2.32 as a mixed number in simplest form.

Answers

Answer:

- 2 8/25

Step-by-step explanation:

Hope this helps! If it does, please mark me brainliest because it will help me. Thank you so much! ;) :)

Answer:

-2.32 as a mixed number is -2 32/100 = -2 8/25

Find the value of x that will make L || M

Answers

Answer:

The value of x = 7 to make lines L and M parallel.

Find the slope
x+3=0

Answers

Answer: undefined

You would subtract 3 from both sides to get x= -3 which would be a vertical line meaning it has no slope

Find the value of each variable (x in this case)

Answers

2x and 4x + 108 are supplementary angles and that means they add up to 180 degrees.

2x + 4x + 108 = 180

Combine like terms

6x + 108 = 180

Subtract 108 from both sides.

6x = 72

Divide both sides by 6

x = 12

If the measures of the angles in the following triangle were listed in increasing order, which measure would be second? (2x+24) ° (4x+43) ° (x^2+1) °

Answers

Answer:

x²+1 is the second

Step-by-step explanation:

2x+24=40°

4x+43=75°

x²+1=65°

(2x+24) + (4x +43) + (x²+1)=180

x²+6x+68=180

x1=-14

x2=8

since there's no negative angle, we'll use x=8

Can someone solve this ASAP!!! pls thx​

Answers

Answer: just add givento number 1 and your all done bro

Step-by-step explanation:

Use the image below to solve for x.

Answers

Answer:

x will equal 61 degrees

Step-by-step explanation:

The total degree of a pentagon is 540. You subtract all the angles you have (138, 144, 107, and 90) and that will get your sum.

PLEASE HELP ITS EASY suppose you are converting x kilograms to ounces. will the number of oz be less than or greater than x?

Answers

Answer:

greater

Step-by-step explanation:

greater beacause 1 kilo =35.274 ounce

The store charges $1.95 for a pack of 5 pens. What is the individual price per pen ?
A - .50
B- $1.95
C- $2.56
D- .39
does anyone know or can explain ?

Answers

The answer is D, 1.95 divided by 5 equals 0.39.

Which of these would not produce a representative sample that determines the favorite sport of the students at the local high school?

Answers

Answer:

The answer is ask ten students wearing football jersey each day for a week

Step-by-step explanation:

Answer:

it its the pizza one because people could be bias and that would not give a very acturate answer

Step-by-step explanation:

6. Evaluate the function at each specified value of the independent variable and simplify. (If an answer is undefined, enter UNDEFINED.) 7. Evaluate the function at each specified value of the independent variable and simplify. (If an answer is undefined, enter UNDEFINED.)

Answers

Complete Question

The complete question is shown on the first uploaded image

Answer:

6a

    [tex]V(3) =  113.112[/tex]

6b

     [tex]V(\frac{3}{2})  = 14.139  [/tex]

6c

   [tex]V(2r) =  33.514r^3[/tex]

7a

  [tex] h(5) = 0 [/tex]

7b

    [tex] h(t) = -5.76 [/tex]

7c

[tex] h(x+5) = x(x+5)[/tex]

Step-by-step explanation:

Considering the question 6

The function given is [tex]V(r) = \frac{4}{3} \pi r^3[/tex]

For  V(3) we have  

        [tex]V(3) =  \frac{4}{3} \pi 3^3[/tex]

       [tex]V(3) =  \frac{4}{3} * 3.142 * 3^3[/tex]

       [tex]V(3) =  113.112[/tex]

For  [tex]V(\frac{3}{2})[/tex]

        [tex]V(\frac{3}{2})  =  \frac{4}{3} *  \pi *   (\frac{3}{2})^3 [/tex]

        [tex]V(\frac{3}{2})  =  \frac{4}{3} * 3.142 *   (\frac{3}{2})^3 [/tex]

        [tex]V(\frac{3}{2})  = 14.139  [/tex]

For   V(2r)

            [tex]V(2r) =  \frac{4}{3} \pi (2r)^3[/tex]

           [tex]V(2r) =  \frac{4}{3} * 3.142*8r^3[/tex]

           [tex]V(2r) =  33.514r^3[/tex]

Considering the question 7

     The function given is  [tex] h(t) = t^2 - 5t [/tex]

For  h(5)

          [tex] h(5) = 5^2 - 5(5) [/tex]    

           [tex] h(5) = 0 [/tex]  

For  h(1.8)

            [tex] h(t) = 1.8^2 - 5(1.8) [/tex]

            [tex] h(t) = -5.76 [/tex]

For  h(x+ 5)

              [tex] h(x+5) = (x+5)^2 - 5(x+5) [/tex]

               [tex] h(x+5) = x^2 + 25+10x-5x-25[/tex]

             [tex] h(x+5) = x(x+5)[/tex]

half a number decreased by three is equal to 38 less than four times the number

Answers

Answer:

10

Step-by-step explanation:

let the unknown number be x

so half of x decreased by 3 is

1/2x -3

It is equal to 38 less than 4 times the number which is x; 4x-37

THEREFORE;

½x-3 =4x -38

x-6/2 = 4x-38

cross-multiply

x-6 =8x -76

x-8x = -76+6

-7x = -70

Divide both sides by -7

So x =10

what is

-4|5+m|
[tex]m = - 3[/tex]

Answers

Steps to solve:

-4|5 + m|

~Substitute

-4|5 - 3|

~Simplify

-4|2|

~Multiply

-8

Best of Luck!

Resolved

1. 2(y+4) +4 (2y+2)



Answers

Answer:

10y+16

Step-by-step explanation:

Distribute and combine like terms

please tell me what x is 3(x+6)=48

Answers

Answer

X=10

Step by step explanation

48÷3 = 16

16-6 = 10

-0.3535 is it rational or irrational​

Answers

Answer:

Rational

Step-by-step explanation:

A rational number is a number expressed as a fraction. So the number could be a decimal or percent.

Hope This Helps :)

The given decimal number is rational number.

What is the rational number?

Rational numbers are in the form of p/q, where p and q can be any integer and q ≠ 0. This means that rational numbers include natural numbers, whole numbers, integers, fractions of integers, and decimals (terminating decimals and recurring decimals).

The given decimal number is -0.3535.

Here, the given decimal number is repeated and terminating decimal

So, the given decimal is rational number

Therefore, the given decimal number is rational number.

To learn more about the rational numbers visit:

brainly.com/question/19161857.

#SPJ2

Taki's are originally $3.25 a bag. You have a 25% off coupon. What
is the final price of the Takis?

Answers

Answer:

3.25 * .75 = $2.44

Step-by-step explanation:

Answer:

Your answer should be $2.44 or around there

Step-by-step explanation:

Why is it imprecise to say that the equation 25 x 2 + 27 + 0 has no solution? Find all solutions of the equation.

Answers

Answer:

77

Step-by-step explanation:

77 is the only correct answer

4 subtracted from twice w, the result is 20 more than w. Find w

Answers

Answer:

w = 24

I hope this helps!

W=12 as shown in pic good luck
Other Questions
In a quiz show, I get 23 out of 50 questions right. Whatpercentage did I get right? Which statement is true about this function for all values of x < 4The function is positive.The function is decreasing.The function is increasing.The function is negative. Write an algebraic expression that represents the following statement: 7 less than the quotient of 5 and n" * The primary purpose of the World Bank is to maintain an orderly system of world trade and exchange rates.TrueFalse Dream act significance on immigration Alebra 1 - 10pts Solve for x.x/4 8A. x2B. x32C. x32D. x2 Please please please help me please I dont know how to do it please help me Can please show all the work please A country's gross domestic product measuresA. the level of education reached by most citizens. the number of different products sold by the country's businessesc. the total number of government construction projects completed each yearD the value of all final goods and services produced within a country in a given yearPlease select the best answer from the choices provided Find the slope of the line between the points (1, 4) and (-1,6) 15. What is Rousseau's point of view? What would Rosseau recommend for your deserted islandgovernment?I After 8 points are added to each score in a sample, the mean is found to be M= 40. What was the value for the original mean Diagram of exhalations Zoe mows the lawn in the summer to earn extra money she usually charges $15 per week but she offers a special rate of $59.50 if a customer prepaid for 5 weeks. how much does a customer pay for each week with a special rate? Are all numbers rational? Which of the following is a promise advertisers could make to promote healthy living using fitness equipment? How can adding dialogue improve a narrative? (Select all correct answers.) Five less than one half of a number is 18.5 = 181/2 5 18x 5 = 185- X = 18 b is the midpoint of AC. a has coordinates (9.11), and B has coordinates (-10,5). find the coordinates of c. find the value of each power 5^-2 The perimeter of a rectangular flower bed is 68 feet. Find the dimensions of the flower bed if the length is 2 less than three times the width.